what is the difference of 3 1/4 and 15/16.

Answers

Answer 1
It is the fact that I don’t know so sorry that u don’t know it either
Answer 2

Answer:

I hope it helps

What Is The Difference Of 3 1/4 And 15/16.

Related Questions

Which equation is equivalent to the formula below?

Which equation is equivalent to the formula below?

Answers

The equivalent equation will be a = (y - k) / (x - h)². Then the correct option is C.

What is an equivalent?

The equivalent is the expression that is in different forms but is equal to the same value.

The definition of simplicity is making something simpler to achieve or grasp while also making it a little less difficult.

The equation is given below.

y = a(x - h)² + k

Simplify the equation for 'x', then we have

y = a(x - h)² + k

y - k = a(x - h)²

a = (y - k) / (x - h)²

The equivalent equation will be a = (y - k) / (x - h)². Then the correct option is C.

More about the equivalent link is given below.

https://brainly.com/question/889935

#SPJ1

2. Write in expanded form
76.345

Answers

Answer:

76.345=

70 +6 +0.3 +0.04 +0.005

explanation:

70 +6 +0.3 +0.04 +0.005= 76.345

I just don’t understand this tried multiple times and looked back on my notes and I can’t seem to solve this

I just dont understand this tried multiple times and looked back on my notes and I cant seem to solve

Answers

Answer:

the answer is 2 fill that in g

Simplified. 2c+2d+5d

Answers

Answer:

2c +7d

Step-by-step explanation:

2c+2d+5d

Combine like terms

2c + d(2+5)

2c +7d

Answer:

The simplified answer of this expression is 2c + 7d

Step-by-step explanation:

For this problem, you will have to combine like terms.

2c + 2d + 5d

Combine 2d and 5d.

2c + 7d

There are two more quarters than dimes and as many nickles as quarters and dimes together. The total amount of money is 3.75. How many quarters, dimes and nickles are there?

Answers

Answer: 9 quarters, 7 dimes, & 16 nickels

Step-by-step explanation:

We set number of dimes as d:

Number of quarters: d + 2Number of nickels: d + 2 + d = 2d + 2

So, set up the equation to solve:

\(0.10d+0.25(d+2)+0.05(2d+2)=3.75\\\\0.10d+0.25d+0.5+0.1d+0.1=3.75\\\\0.45d=3.75-0.5-0.1=3.15\\\\d=7\)

So now that we know there're seven dimes, we can plug them in our initial equations to find the number of quarters and nickels:

Number of quarters: d + 2 = 7 + 2 = 9Number of nickels: d + 2 + d = 2d + 2 = 2(7) + 2 = 16

does tan^2x = sin^2x/cos^2x

Answers

Answer:

Yes they both are equal

the monthly rental for an apartment is 412.50 how much would the rent be for a year?

Answers

Answer:

4950

Step-by-step explanation:

412.50 x 12

412.50 is the mothly rental

12 are the month in a year

so it is 4950

Barry wants to open a checking account. He wants online services, and a debit card. He is looking at the following tables of fees from his employer's credit union, EECU, and a local bank, E-town Bank:
A 4-column table with 5 rows. Column 1 is labeled E E C U with entries monthly fee, non-sufficient funds fee, second copy of statement, network A T M usage, online services and debit cards. Column 2 is labeled Fee Table with entries 0 dollars, 28 dollars, 2 dollars, 2 transactions per month free after that 2 dollars each, free. Column 3 is labeled E-town Bank with entries monthly fee, non-sufficient funds fee, replacement statement copy, Network A T M usages, online services and debit cards. Column 4 is labeled Brief schedule of common fees with entries 2 dollars and 50 cents, 30 dollars, 5 dollars, 2 dollars each, free.
Based upon past experience, Barry expects no overdrafts. He expects no 2nd copies of statements. Barry estimates that he will use network ATMs about 5 times a month with either bank. Barry decides in the end to choose EECU. Assuming that both banks provide the necessary services equally well, and based upon the tables of fees given above, how much can Barry reasonably expect to save annually by choosing EECU in this case over E-town bank?
a.
$72
b.
$78
c.
$144
d.
$24

Answers

Answer:

i just want these points

Step-by-step explanation:

a box (with no top) is to be constructed from a piece of cardboard of sides and by cutting out squares of length from the corners and folding up the sides as in the figure below: suppose that the box height is in. and that it is constructed using 136 of cardboard (i.e., ). which values and maximize the volume?

Answers

The values that maximize the volume  for the given height 2 in. using 136 square inches cardboard is given by A = √120 and B = √120.

Height of the box = 2 in.

Box constructed using 136 in.2 of cardboard

To maximize the volume of the box, determine the values of A and B that will result in the maximum volume.

Let begin by calculating the dimensions of the box.

Rectangular piece of cardboard with sides A and B, and we cut out squares of length h from each corner.

Then, fold up the sides to form the box.

The dimensions of the cardboard before cutting are,

Length = A

Width = B

Height = h

           = 2 in.

After cutting and folding, the box dimensions become,

Length = A - 2h

            = A - 4 in.

Width = B - 2h

         = B - 4 in.

Height = h

           = 2 in.

Now, calculate the volume of the box using these dimensions,

Volume = Length × Width × Height

Volume = (A - 4) × (B - 4) × 2

The box is constructed using 136 in² of cardboard,

which means the area of the cardboard used is 136 in².

The area of the cardboard before cutting is equal to the area of the box plus the areas of the cut-out squares.

⇒Area of the cardboard = Area of the box + 4 × Area of the cut-out squares

⇒Area of the cardboard = A × B + 4 × (2²)

Since the area of the cardboard is given as 136 in², we have,

⇒A × B + 4 × (2²) = 136

⇒A × B + 16 = 136

⇒A × B = 120

We now have two equations,

A× B = 120

Volume = (A - 4) × (B - 4) × 2

To find the values of A and B that maximize the volume,

Express one variable in terms of the other using the equation A × B = 120.

Solve this equation for A,

⇒A = 120 / B

Substituting this into the volume equation, we have,

⇒Volume = (120 / B - 4) × (B - 4) × 2

⇒Volume = (120B -4B² + 16B -480 )/B × 2

⇒Volume = (-4B² + 136B + 480)/B × 2

⇒Volume = (-8B² + 272B - 960) /B

⇒Volume = (-8B + 272 - 960/B )

Now, find the maximum value of the volume,

Differentiate the volume equation with respect to B and set it equal to zero to find the critical points,

dV/dB = -8 + 960/B²

dV/dB = 0

⇒-8 + 960/B² = 0

⇒ B = √120 or

The maximum volume occurs when B takes on its maximum possible value, which is when B = √120.

Substituting this value of B back into the equation A × B = 120, we can solve for A,

A×√120 = 120

⇒A = 120 / √120

⇒A = √120

Therefore, the values that maximize the volume are A = √120 and B = √120.

Learn more about volume here

brainly.com/question/2140766

#SPJ4

The above question is incomplete , the complete question is:

A box (with no top) is to be constructed from a piece of cardboard of sides A and B by cutting out squares of length h from the corners and folding up the sides as in the figure below:

Image for A box (with no top) is to be constructed from a piece of cardboard of sides A and B by cutting out squares of

Suppose that the box height is h=2 in. and that it is constructed using 136 in.2 of cardboard (i.e., AB=194). Which values A and B maximize the volume?

If (2,1) is the midpoint of the line segment ST and the coordinates of S are (5,4), find the coordinates of T.

Answers

Answer:

Step-by-step explanation:

The formula to find the midpoint of a segment is

\(M=(\frac{x_1+x_2}{2},\frac{y_1+y_2}{2})\) and we have the midpoint and we also have a coordinate of (5, 4). Let's let x₁ = 5 and y₁ = 4. Filling in what we have:

\((2,1)=(\frac{5+x_1}{2},\frac{4+y_2}{2})\) and we'll deal with the x terms first. The x coordinate of the midpoint is 2, so:

\(2=\frac{5+x_2}{2}\) and multiply both sides by 2 to get rid of the denominator to get:

4 = 5 + x₂ so

x₂ = -1. Going on to the y coordinate. The y coordinate of the midpoint is 1, so:

\(1=\frac{4+y_2}{2}\) and again multiply both sides by 2 to get rid of the denominator to get:

2 = 4 + y₂ so

y₂ = -2

The coordinates of T are (-1, -2)

Tom bought a bag full of coal. After one week the bag was 2|3 full. During the next week he used of the remaining coal What fraction was left in the bag?​

Answers

Answer:

None

Step-by-step explanation:

When he first bought the coal, it was full. After one week, he used 1/3, leaving the bag 2/3 full. If he used the remaining coal in the bag, that means he used all of the coal - all two thirds of it.

The fraction of coal left in the bag was zero or none since Tom used the remaining coal.

What is the reasoning?

The procedure of utilizing logical reasoning to analyze a condition to determine the best problem-solving approach for a particular issue, then using this approach to create and explain a resolution.

The reasoning is a tricky but interesting problem in mathematics that requires relations of variables to solve.

As per the given,

After one week the bag was 2|3 full

Now remaining coal = 1 -2/3 = 1/3 part

Now, Tom used this remaining amount.

So, 1/3 - 1/3 = 0 part.

Hence "The fraction of coal left in the bag was zero or none since Tom used the remaining coal".

For more about reasoning,

https://brainly.com/question/2923977

#SPJ2

A streaming video service administers a survey through its platform to evaluate the variety of options provided through the service. This is one of the questions on the survey.

Rate the variety of material available through our service on a scale of 1 to 4 stars, with 4 being the highest.

☆ ☆ ☆ ☆

Which of these could affect the results of the survey?

A.
The survey is biased because it was administered through the service instead of in person.
B.
The survey is biased because it is being taken only by the service’s users.
C.
People who don’t have the service can’t take the survey.
D.
The answer options could be interpreted differently by different users.

Answers

Answer:

The answer for you here is D.

Step-by-step explanation:

A. makes no sense, I don't think I need to explain myself there lol.

B. and C. both pretty much say the same thing, since you can only pick one that makes those answers sketchy. Not to mention, if anything it would be MORE biased for the survey to allow people who don't even use the service to vote.

D. This one simply makes the most sense, as that is indeed something that varies greatly from person to person. (Also I got this one right on the test :P   )

The option that could affect the results of the survey is the answer options could be interpreted differently by different users.

What can affect the results of the survey?

The survey is meant to be taken only by those who use the service. The fact that the survey administered through the service instead of in person does not make the survey biased.

The question is too ambiguous and it can be interpreted differently by the users.

To learn more about surveys, please check: https://brainly.com/question/13532910

#SPJ2

Please help!

You flip a coin twice.

What is the probability of getting tails and then getting heads?

Write your answer as a percentage.


___%

Please help!You flip a coin twice.What is the probability of getting tails and then getting heads?Write

Answers

Answer: 25%

Step-by-step explanation:

You have a 1/2 chance of landing a heads or tails both times. So, you multiply 1/2 *1/2 = 1/4. 1/4 = 0.25 = 25%.


The exterior angles of a hexagon are 56º, 730, 2xº, 3x°, 4x° and 5xº.
Find the value of x.

Answers

Answer:

x = 16.5

Step-by-step explanation:

im assuming you meant 73 degrees instead of 730.

56+73 = 129

2x+3x+4x+5x = 14x

360-129 = 231

231 = 14x

231/14 = 16.5

x = 16.5

If the Cable Company offers cable for $110 a month but gives a 10% discount for new customers. Find the cost for the new customers.

Answers

The cost for new customers who are entitled to the 10% discount is $99 per month.

The Cable Company offers cable for $110 a month but gives a 10% discount for new customers.

To find the cost for new customers, we will have to subtract the 10% discount from the original cost of $110 per month.

So, we will have to multiply the original cost by the percentage of the discount which is 10%.10% of 110 = (10/100) * 110= 11

Therefore, the discount offered by the company is $11.

Now, we will have to subtract the discount from the original cost:

Cost for new customers = $110 - $11 = $99

To know more about percentage, visit:

https://brainly.com/question/32197511

#SPJ11

Is the graph a linear function?
(Enter a math answer)

Is the graph a linear function?(Enter a math answer)

Answers

Yes the graph is a linear function. I don’t really understand what to do now so I figured I’m going to show you the equation is well: y = -2x + 2

It is the parallelogram that has its unequal contiguous sides and the four right angles. A) Square B) Rhombus C) Rectangle D) Rhomboid

Answers

Answer:

Rhomboid is the parallelogram that has its unequal contiguous sides and the four right angles

Hope it helps you

Mark it as brainliest and follow me

My account you can see here below

(hardik 60)

An ice field is melting at a rate of M(t)=4-sin^3 t acre-feet per day. How many acre feet of this ice field will melt from the beginning of day 1 (t=0) to the beginning of day 4 (t=3).

Answers

the amount of ice that will melt from the beginning of day 1 to the beginning of day 4 can be found by integrating the rate of melting over that time period.


To find the amount of ice that melts over the time period from t=0 to t=3, we need to integrate the given rate of melting function, M(t)=4-sin^3 t, over that time period. Using the fundamental theorem of calculus, we can find the antiderivative of M(t):

∫M(t)dt = ∫(4-sin^3 t)dt = 4t + (3/4)cos(t) + (1/12)cos^3(t)

Evaluating this antiderivative from t=0 to t=3, we get:

(4(3) + (3/4)cos(3) + (1/12)cos^3(3)) - (4(0) + (3/4)cos(0) + (1/12)cos^3(0))

Simplifying this expression, we get:

12 + (3/4)cos(3) + (1/12)cos^3(3) - (3/4)

Therefore, the amount of ice that will melt from the beginning of day 1 to the beginning of day 4 is approximately 11.56 acre-feet.

we can find the amount of ice that will melt over a given time period by integrating the rate of melting function over that time period. In this case, we found that approximately 11.56 acre-feet of the ice field will melt from the beginning of day 1 to the beginning of day 4.

To know more about time period, visit:

https://brainly.com/question/31824035

#SPJ11

When planting a garden, you can choose from five types of tomatoes, two types of peppers, and four types of squash. If you choose only one crop from each category, how many possible garden combinations can you make?


When building a computer, you need to make several choices, such as type of storage, level of memory, and type of processor. You can choose from three types of storage, six levels of storage, and four types of processor. Calculate the number of combinations that you have if these are the only three choices you need to make.

Answers

The main answer is as follows:For the question, "When planting a garden, you can choose from five types of tomatoes, two types of peppers, and four types of squash. If you choose only one crop from each category, how many possible garden combinations can you make?"

There are five types of tomatoes, two types of peppers, and four types of squash to choose from. The number of possible combinations is calculated using the multiplication principle because there are independent options. The multiplication principle asserts that if there are m ways to accomplish one event and n ways to accomplish a second event, there are m × n ways to accomplish both events.For this question, we can determine the number of combinations by multiplying the number of choices in each category. So the number of possible combinations is 5 x 2 x 4 = 40For the question, "When building a computer, you need to make several choices, such as type of storage, level of memory, and type of processor. You can choose from three types of storage, six levels of storage, and four types of processor. Calculate the number of combinations that you have if these are the only three choices you need to make."There are three types of storage, six levels of storage, and four types of processors to choose from. As a result, the number of possible combinations is calculated by multiplying the number of options available for each category. So, the number of possible combinations is 3 x 6 x 4 = 72

:Given:Types of tomatoes = 5Types of peppers = 2Types of squash = 4To find:How many possible garden combinations can you make if you choose only one crop from each category?Solution:According to the multiplication principle, if there are m ways to accomplish one event and n ways to accomplish a second event, there are m × n ways to accomplish both events.So, using the multiplication principle, we can determine the number of combinations by multiplying the number of choices in each category.The number of possible garden combinations = Number of types of tomatoes × Number of types of peppers × Number of types of squash= 5 × 2 × 4= 40Hence, there are 40 possible garden combinations if you choose only one crop from each category.Given:Types of storage = 3Levels of storage = 6Types of processors = 4To find:How many possible combinations you have if these are the only three choices you need to make?Solution:According to the multiplication principle, if there are m ways to accomplish one event and n ways to accomplish a second event, there are m × n ways to accomplish both events.So, using the multiplication principle, we can determine the number of combinations by multiplying the number of options available for each category.Number of possible combinations = Number of types of storage × Number of levels of storage × Number of types of processors= 3 × 6 × 4= 72Hence, there are 72 possible combinations if these are the only three choices you need to make.

To know more about combinations visit:

https://brainly.com/question/20211959

#SPJ11

HELP ASAP!!! PLS PLS!! NO WRONG ANSWERS PLZ

HELP ASAP!!! PLS PLS!! NO WRONG ANSWERS PLZ

Answers

Answer:

(4,2)

Step-by-step explanation:

To find the solution, we can simply find the intersection. That is (4,2)

Answer:

D

Step-by-step explanation:

because both lines meet at point (4,2)

HELP PLEASE I appreciate it

HELP PLEASE I appreciate it

Answers

Answer:

80 degrees

Step-by-step explanation:

Answer:x=37

Step-by-step explanation:

2x+1=75, then you subtract 1 from each side, which will give you 2x=74, then you divide both by 2 and get x=37



If a has the given length, how many triangles can be formed? (Hint: Rotate the strip to see if the mark can intersect the ray at any other locations to form a different triangle.)

Answers

For the length of one side, "a," it can be form an infinite number of triangles using that side length.

In order to construct a triangle, we need at least three sides, and their lengths must satisfy the triangle inequality theorem.

The triangle inequality theorem states that the sum of the lengths of any two sides of a triangle must be greater than the length of the remaining side.

Given the length of side "a," choose any two additional side lengths that satisfy the triangle inequality theorem to form a triangle.

Since there are infinite possibilities for selecting the lengths of the other two sides as long as they satisfy the inequality,

conclude that an infinite number of triangles can be formed.

Therefore, for 'a' as side length infinite number of triangles are possible.

Learn more about triangles here

brainly.com/question/11961976

#SPJ4

$1.77. Lian weighs a bag of apples on the
Store's Scales. The scale shows that the appies
weigh 1.32 lb. How much do lian's apples
a grocery store, each pound of apples Costs
Cost? show your work.

Answers

Answer:

About $2.34

Step-by-step explanation:

If each pound of apples cost 1.77 and you have 1.32 pounds, you would multiply 1.77 by 1.32 to get 2.3364, but when rounded, 2.34.

Answer:

Step-by-step explanation:

1.77 answer 2.34

or somthing like that

could someone help with this

could someone help with this

Answers

∠1 = 26°

2 = 154°

3 = 26°

4 = 26°

5 = 154°

6 = 154°

7 = 26°

Step-by-step explanation:

So, we have two parallel lines cut by a transversal (C).

Vertical angles are always equal.

∠2 = 154° (vertical angles)

Linear pair make up 180°.

∠1 = 180° - 154° = 2 (linear pair)

∠1 = ∠3 = 2 (vertical angles)

Corresponding angles are always equal.

∠5 = 154° (corresponding angles)

∠5 = ∠6 = 154° (vertical angles)

Alternate Interior angles are always equal.

∠3 = ∠4 = 2 (alternate interior angles)

∠4 = ∠7 = 2 (vertical angles)

During its first month of business, Dig the Dogs, Inc. purchased $900 of hotdogs of which it paid $300 and owes the rest. During the month, it sold 3/4 of its inventory for $1,000 on account. What is the amount of Cost of Goods Sold for the month ended?

Answers

Cost of goods sold (COGS) is an important accounting calculation that measures the costs associated with selling a company’s goods or services. Therefore, the cost of goods sold for the month ended is $600.

Cost of Goods Sold (COGS) is a measure of the cost of goods sold during the accounting period. It is calculated by subtracting the beginning inventory from the cost of goods purchased during the accounting period and then adding the ending inventory. For Dig the Dogs, Inc., the cost of goods sold for the month ended is calculated as follows: Beginning Inventory = 0 Cost of Goods Purchased = $900 Ending Inventory = $300 .COGS = Beginning Inventory + Cost of Goods Purchased - Ending Inventory  = 0 + 900 - 300   = $600 Therefore, the cost of goods sold for the month ended is $600.

Learn more about cost here:

https://brainly.com/question/19075809

#SPJ4

Which expression is equivalent to (m−2n−3)−4?

m−6n−7
m8n12
m−8n−12
1 over the quantity m raised to the sixth power times n raised to the seventh power end quantity

Answers

Answer:

B

Step-by-step explanation:

(m^-2 n^-3)^-4

-2 x -4 = 8

-3 x -4 = 12

m^8n^12

The given expression (m − 2n − 3) − 4 is equivalent to →

(m - 2n - 7).

What is a mathematical function, equation and expression?function : In mathematics, a function from a set X to a set Y assigns to each element of X exactly one element of Y. The set X is called the domain of the function and the set Y is called the codomain of the function.expression : A mathematical expression is made up of terms (constants and variables) separated by mathematical operators.equation : A mathematical equation is used to equate two expressions.

Given is the expression -

(m − 2n − 3) − 4

The given expression is -

(m − 2n − 3) − 4

m - 2n - 3 - 4

m - 2n - 7

Therefore, the given expression (m − 2n − 3) − 4 is equivalent to →

(m - 2n - 7).

To solve more questions on functions, expressions and polynomials, visit the link below -

brainly.com/question/17421223

#SPJ2

PLEASE HELP I NEED HELP

Which verb of ser do I use?

Example: son, sois, somos, es, eres, soy

Ricardo y yo _____.

I WILL GIVE BRAINLIEST

Answers

Answer: Ricardo y yo somos

Answer:

somos

Step-by-step explanation:

somos means us

Ricardo and us

Can y’all please help me

Can yall please help me

Answers

Answer:

Rly lazy man.....

Step-by-step explanation:

1/2*l*w is the area of triangle, plug in values...

1/2*4.5*6.6

14.85 square feet is the area

Answer:

A=14.85

Step-by-step explanation:

\(A=\frac{ab}{2}\)

"ab' are the lengths of the sides

\(A=\frac{(4.5)(6.6)}{2} \\\\A=\frac{29.7}{2} \\\\A=14.85\)

Hope this helped and answered your question! Have a great day/night!

two fan blades in the shape of equilateral triangles are rotates clockwise around point p. how many degrees will the blades have rotates when vertex d reaches the point where vertex c is nwo

Answers

The fan blades will have rotated 180 degrees when vertex D reaches the point where vertex C is now.

When the two fan blades in the shape of equilateral triangles rotate clockwise around point P, the vertex D will reach the point where vertex C is now.
To determine the number of degrees the blades will have rotated, we need to know the number of degrees in a full rotation. A full rotation consists of 360 degrees.
Since both blades rotate together, they will cover the same number of degrees. Therefore, when vertex D reaches the point where vertex C is now, the blades will have rotated by half of a full rotation, which is 180 degrees.

In summary, the fan blades will have rotated 180 degrees when vertex D reaches the point where vertex C is now.

To know more equilateral triangles, click here

https://brainly.com/question/12990950

#SPJ11

What is the probability of NOT landing on a vowel when an alphabetical dice is rolled?

What is the probability of NOT landing on a vowel when an alphabetical dice is rolled?

Answers

Answer:

An alphabetical dice has 6 sides labeled with the letters A, B, C, D, E, and F. Of these, A, E, and I are vowels and the remaining letters are consonants.

The probability of landing on a vowel on any given roll is therefore 3/6 = 1/2.

The probability of NOT landing on a vowel on any given roll is therefore 1 - 1/2 = 1/2.

Therefore, the probability of NOT landing on a vowel when an alphabetical dice is rolled is 1/2 or 0.5.

Step-by-step explanation:

Other Questions
which statement ensures that a file will be closed even when an exception is raised? Consider the following geometry problems in 3-space Enter T or F depending on whether the statement is true or false. (You must enter T or F.. True and False will not work.) 1. Two planes orthogonal to a third plane are parallel 2. Two lines parallel to a plane are parallel 3. Two planes parallel to a third plane are parallel 4. Two planes parallel to a line are parallel Hello im in 8th grade and I need to get my grades up so anyone know the answer to -12 minus 12x = 36work also needs to be shown if possible thank you A stock produced annual returns of 8 percent, -12 percent, 6 percent, 1 percent, and 19 percent over the last five years, respectively. What is the variance of these returns?a) .044667b) .012730c) .009914d) .010184e) .050920 Which of the following exists within the social media policy and ensures that readers/consumers can still find postings credible and trustworthy?a. Social media guidelines.b. Social media policy.c. Standards of conduct.d. Disclosure requirements.e. None of these. The phosphorous, P2O5, in a 47. 683 g sample of plant food was converted to PO4 3- through a hydrolysis reaction and then precipitated as Ag3PO4 through the addition of 50. 00 mL of 0. 9205 M AgNO3. The excess AgNO3 was back-titrated with 16. 24 mL of 0. 2007 M KSCN. From this information determine the wt% P2O5 in the plant food sample Which native american group is know for an organized, chiefdom, sedentary society that relied on agriculture and hunting for their food sources?. Danny charges $75 for 5 hours of swimming lessons, Martin charges $72 for 4 hours of swimming lessons. Who offers a better deal? Natural gas definition which of the following school policies is most likely to have a positive impact on the digital divide? responses a school allows students to bring a graphing calculator from home to complete in-class mathematics assignments. a school allows students to bring a graphing calculator from home to complete in-class mathematics assignments. a school allows students to bring a tablet computer to class every day to participate in graded quizzes. a school allows students to bring a tablet computer to class every day to participate in graded quizzes. a school provides a laptop or tablet computer to all students enrolled at the school. a school provides a laptop or tablet computer to all students enrolled at the school. a school recommends that all students purchase a computer with as much processing speed as possible so that projects run faster. A 30-year-old man presents with asymmetric myalgias and arthralgias. He also complains of difficulty climbing stairs. You note fever, hip, and shoulder muscle weakness but no atrophy, scattered extremity numbness, and tender palpable purpura. However, there is no facial or truncal rash. Laboratory testing reveals a low hematocrit, a high creatinine kinase, a negative antinuclear antibody titer, and an elevated erythrocyte sedimentation rate. Which of the following is the most likely diagnosis? which of the following descriptions accurately characterizes persuasive messages? check all that apply. they must be conveyed in writing. they can be verbal or nonverbal. they often appeal to our emotions. they are effective only when using rational appeals. In kite WALT, mELA=48. Identify mWAL.mWAL = 42mWAL = 84mWAL = 48mWAL = 90 What is the name of the island where the original jurassic park is built?. Mai gave half of her brownies, and then half a brownie more, to Kiran. Then she gave half of what was left, and half a brownie more, to Tyler. That left her with one remaining brownie. How many brownies did she have to start with? Select the statements that explain marking and categorizing messages. Check all that apply.The Read/Unread option is a toggle button that marks a message read or unread.You can mark a message as unread when you want to come back to it later.You can mark a message as read by turning off the Categories option.Categories are color coded and can be renamed and sorted.Grouping different types of messages together can be done using Read/Unread.answer: A B D Name the coefficient in the following equation. 8 = 2y + 3 which of part of the marketing plan is a summary of the key points and should be completed after the rest of the marketing plan has been written? What does the pair of shoes weigh on earth? What are the two primary purposes that interest groups and organizations aim for when communicating with members through email?Interest groups use email lists to (demonstrate, inform, or attend) and make (demands on, criticisms of, or asks of) their members.